4
$\begingroup$

Let $f^N_n: [0,T]\to [0,2]$ be continuous functions such that for all $n=1,\ldots, N-1$

$$f_n^N(t) \le Cn\int_0^t \big(f_{n+1}^N(u)+f_n^N(u)\big)du + \frac{Cn^2}{\sqrt{N-n}}t,\quad \forall t\in [0,T],$$

where $C>0$ denotes some constant. Assume that $f^N_n(0)=0$. Can we prove that for every fixed $n$ and $t$

$$\lim_{N\to\infty} f^N_n(t)=0?$$

$\endgroup$
11
  • $\begingroup$ Is there any reason we need to "assume" that $f^N_n(0)=0$ when $n<N$ ? I think $f^N_n(0)=0$ comes from your inequality : $f^N_n(0)\le 0+0$ because first term is an empty integral, and second term is multiplied by $t=0$. And when $n=N$, I'm not sure having $f^N_N(0)=0$ or not makes a huge difference, because $f^N_N(t), t>0$ can be arbitrary big, as long as it remains continuous and in $[0,2]$ (I mean we can have $f^N_N(\epsilon)=2$ for $\epsilon > 0$ arbitrary small). $\endgroup$ Commented Nov 17 at 12:10
  • $\begingroup$ @ewen-goisot Indeed the sequence $f^N_n$ is uniquely defined for $1\le n\le N$. You are absolutely right this inequality yields $f^N_n(0)=0$ for $1\le n<N$. $\endgroup$ Commented Nov 17 at 12:42
  • $\begingroup$ is my proof (in my answer) clear now, or should I add something else ? $\endgroup$ Commented Nov 28 at 10:45
  • 1
    $\begingroup$ @ewen-goisot - $F^M_n(t)=\min\big(2,\quad Cn\int_0^t F^M_n(u)+F^M_{n+1}(u)du\big), \quad\forall T \in \mathbb{R}^+, \quad n\le M-1$. Maybe you can justify $F^M_n$ is well defined implicitely? - $G^M_{M-1}(t)=4Cnt, \quad\forall t \in [0,2T^M_{M-1}]$. What is $n$? Do you refer to $n=M-1$? $\endgroup$ Commented Nov 29 at 9:02
  • 1
    $\begingroup$ @ewen-goisot Many thanks. I will get back to you as soon as I understand the proofs of Propositions 1 and 2 $\endgroup$ Commented Dec 1 at 10:29

1 Answer 1

0
$\begingroup$

Slightly simplifying the problem

$t=0$ implies $f^N_n(t)=0$ for any n<N :

We can prove that for any $n<N$, $f^N_n(t)=0+0$ because it's the sum of an empty integral and something multiplied by $t=0$.

We don't need $T$ :

Let's assume that the $f^N_n$ are satisfying the inequality in the interval $[0,T]$.

Then we can define functions $g^N_n$ by :

  • $g^N_n(t)=f^N_n(t), \quad\forall t \in [0,T]$ and
  • $g^N_n(t)=f^N_n(T), \quad\forall t \in [T,+\infty[$.

Then, let $t\in [T,+\infty[, n<N\in ℕ$. $g(t)=f(T)=g(T)\le Cn\int_0^T \big(g_{n+1}^N(u)+g_n^N(u)\big)du + \frac{Cn^2}{\sqrt{N-n}}T$ $\le Cn\int_0^t \big(g_{n+1}^N(u)+g_n^N(u)\big)du + \frac{Cn^2}{\sqrt{N-n}}t$ because all the terms are positive.

Then, for simplicity, we can assume that the $f_n^N$ are from $\mathbb{R}^+$ to $[0,2]$.

Upper bound focusing on the integrals

Here, I write $M$ instead of $N$ on purpose, because I need both for next part.
Let's define some other functions $F^M_n:\mathbb{R}^+\to[0,2]$ similar to $f$ :

  • $F^M_M(t)=2, \quad\forall t \in \mathbb{R}^+$
  • $F^M_n(t)=\min\big(2,\quad Cn\int_0^t F^M_n(u)+F^M_{n+1}(u)du\big), \quad\forall T \in \mathbb{R}^+, \quad n\le M-1$.

Let's now create another set of functions $G^M_n:\mathbb{R}^+\to[0,2]$. The $G^M_n$ are designed to be upper bounds to the $F^M_n$ (not directly to the $f^M_n$, but we will care about it later), and with more easily computable values (we use multiplications instead of recursive integrals). For $M\in\mathbb{N}$ :

  • $T^M_M=0, \quad T^M_{M-1}=\frac{1}{4C(M-1)}$.
  • $T^M_n=T^M_{n+1}+\frac{1}{8Cn}, \quad\forall n\le M-2$.
  • Note that $(T^M_n)_n$ is strictly decreasing.
  • $G^M_M(t)=2, \quad\forall t \in \mathbb{R}^+$.
  • $G^M_{M-1}(t)=4C(M-1)t, \quad\forall t \in [0,2T^M_{M-1}]$.
  • $G^M_{M-1}(t)=2, \quad\forall t \in [2T^M_{M-1},+\infty[$.
  • $G^M_n(t)=\frac{1}{2}G^M_{n+1}(t), \quad\forall t \in [0,T^M_{n+1}], \quad n\le M-2$.
  • $G^M_n(t)=\frac{1}{2}+4Cn(t-T^M_{n+1}), \quad\forall t \in [T^M_{n+1}, T^M_{n+1}+3(T^M_n-T^M_{n+1})], \quad n\le M-2$.
  • $G^M_n(t)=2, \quad\forall t \in [T^M_{n+1}+3(T^M_n-T^M_{n+1}),+\infty[, \quad n\le M-2$.

Now, let's observe a few properties functions $G^M_n$ have :

  • $G^M_n(T^M_M)=G^M_n(0)=0, \quad\forall n\le M-1$.
  • $G^M_{M-1}(T^M_{M-1})=\frac{4C(M-1)}{4C(M-1)}=1$.
  • $G^M_n(T^M_n)=G^M_n(T^M_{n+1}+(T^M_n-T^M_{n+1}))=\frac{1}{2}+\frac{4Cn}{8Cn}=1, \quad\forall n\le M-2$.
  • By induction, $G^M_n(T^M_m)=\frac{1}{2^{m-n}}, \quad\forall n\le m\le M-1$.
  • For $n\le M-1$, on each interval $[T^M_{m+1}, T^M_m], m\le n$ and $[T^M_n, T^M_n+2(T^M_n-T^M_{n+1})]$ and $[T^M_n+2(T^M_n-T^M_{n+1}),+\infty[$, $G^M_n$ is an affine function. In other words, $G^M_n$ is piecewise affine.
  • For any $n$, $t\mapsto G^M_n(t)$ is a continuous non-decreasing function.

Proposition 1 :
$G^M_n(t)\ge \min\big(2,\quad Cn\int_0^t G^M_n(u)+G^M_{n+1}(u)du\big), \quad\forall t \in \mathbb{R}^+, \quad n\le M-1$.
When $G^M_n(t)=2$, the result is directly true. Let's focus on the case where $G^M_n(t)<2$, i.e. $t\in [0,T^M_n+2(T^M_n-T^M_{n+1})[$.

Proof for $n=M-1$ :
Let $t\in [0,T^M_{M-1}]$, then $G^M_{M-1}(t)=4Cnt=Cn\int_0^t 2+2du \ge Cn\int_0^t G^M_M(u)+G^M_{M-1}(u)du$ because we always have $G^M_n(t)\le 2$. $\quad\square$

Proof for $n\le M-2$ :
We well prove it on intervals of the form $[T^M_{M-k}, T^M_{M-k-1}]$, by induction on $k$.

Initialization :
When $k=0$, we mimic the proof given for $n=M-1$ : Let $t\in [T^M_M,T^M_{M-1}]=[0,T^M_{M-1}]$.
$G^M_n(t)=\frac{4Cnt}{2^{(M-1)-n}} =Cn\int_0^t\frac{1}{2^{(M-1)-(n+1)}}+\frac{1}{2^{(M-1)-(n+1)}}du$ $\ge Cn\int_0^t\frac{1}{2^{(M-1)-(n+1)}}+\frac{1}{2^{(M-1)-n}}du = Cn\int_0^t G^M_{n+1}(T^M_{M-1})+G^M_n(T^M_{M-1})du\quad(*)$ $\ge Cn\int_0^t G^M_{n+1}(u)+G^M_n(u)du$, because the $G^M_m$ are non-decreasing.

$(*)$ This inequality is actually strict when $t>0$.
We will need this point later, on the proof of proposition 2.

Heredity :
Let's assume that, for some $k\le M-n-2$, we have $G^M_n(T^M_{M-k})\ge Cn\int_0^{T^M_{M-k}} G^M_n(u)+G^M_{n+1}(u)du$. Let $t\in [T^M_{M-k}, T^M_{M-(k+1)}]$. $G^M_n(t)=G^M_n(T^M_{M-k})+\frac{t-T^M_{M-k}}{T^M_{M-(k+1)}-T^M_{M-k}}\times \big(G^M_n(T^M_{M-(k+1)})-G^M_n(T^M_{M-k})\big)$ $=G^M_n(T^M_{M-k})+\frac{t-T^M_{M-k}}{1/(8C(M-(k+1)))}G^M_n(T^M_{M-k})$ $\ge G^M_n(T^M_{M-k})+(t-T^M_{M-k})(8Cn)G^M_n(T^M_{M-k})$ $\ge G^M_n(T^M_{M-k})+(t-T^M_{M-k})Cn(G^M_n(T^M_{M-(k+1)})+G^M_{n+1}(T^M_{M-(k+1)}))$ $\ge Cn\int_0^{T^M_{M-k}} G^M_n(u)+G^M_{n+1}(u)du \quad + \quad Cn\int_{T^M_{M-k}}^t G^M_n(u)+G^M_{n+1}(u)du$.
The induction hypothesis is used in the last inequality. $\quad\square$

Heredity when $k=M-n-1$:
Let $t\in [T^M_{n+1},T^M_n+2(T^M_n-T^M_{n+1})]$. $G^M_n(t)=\frac{1}{2}+4Cn(t-T^M_{n+1}) =G^M_n(T^M_{n+1})+Cn\int_{T^M_{n+1}}^t 2+2 du$ $\ge Cn\int_0^{T^M_{M-k}} G^M_n(u)+G^M_{n+1}(u)du \quad + \quad Cn\int_{T^M_{M-k}}^t G^M_n(u)+G^M_{n+1}(u)du$.
The induction hypothesis is used (again) in the last inequality. $\quad\square$

Once the behavior of $G$ is understood, we can easily compare it to $F$.

Proposition 2 :
$F^M_n(t)\le G^M_n(t), \quad\forall M, n, t$.
The equality holds only when $t=0$ or $M=n$ or $G^M_n(t)=2$.

Proof :
By contradiction, let $n$ be the biggest number the proposition is wrong. By $(*)$, we know that there is $\epsilon>0$ such that $\forall t\in ]0,\epsilon], G^M_n(t)>F^M_n(t)$.

Let $t>0$ be the smallest number where $F^M_n(t)=G^M_n(t)<2$. Actually, $t$ should be the "inf" not the "min", but here it's actually the same because both $F^M_n$ and $G^M_n$ are continuous.

By proposition 1, $G^M_n(t)\ge Cn\int_0^t G^M_n(u)+G^M_{n+1}(u)du$
$> Cn\int_0^t F^M_n(u)+F^M_{n+1}(u)du = F^M_n(t)$ (the last inequality is because $t$ is minimal).
That's a contradiction. $\quad\square$

Now, we can prove that the limit is zero. Not already for $f$ but at least for $F$, using an harmonic serie.

Proposition 3 :
$lim_{M\to +\infty}F^M_n(t)=lim_{M\to +\infty}G^M_n(t)=0, \quad\forall t\in\mathbb{R}^+,\quad n\in\mathbb{N}$.

Proof
Let $t\in\mathbb{R}^+, n\in\mathbb{N}, C>0$. As long as $0\le F^M_n(t)\le G^M_n(t)$, we only need to prove the result for $G$.

Let's find an estimation of $T^M_n$.
$T^M_n=T^M_{M-1}+\sum_{i=n}^{M-2}(T^M_i-T^M_{i-1}) =\frac{1}{4C(M-1)}+\sum_{i=n}^{M-2}\frac{1}{8Cn} =\frac{\log(M)-\log(n)+O(1)}{4C}$.
In particular, it implies that $\lim_{M\to +\infty}T^M_n=+\infty$.

We can take $T^M_n$ big enough to be greater than $t$. $G^M_n(T^M_n)=1$ by construction, and $G^M_n$ is convex on $[0,T^M_n]$ ($G^M_n$ is differentiable almost everywhere, and $G^{M'}_n$ is non-decreasing where it's defined). So, $G^M_n(T^M_n)\le\frac{t}{T^M_n}\to 0$ because $T^M_n\to+\infty$. $\quad\square$

Upper bound for the second term

$f^N_n$ is defined almost the same way $F^M_n$ is, except that it has a second term $\frac{Cn^2}{\sqrt{N-n}}t$. When $n<<N$, those terms are very small.

Let's assume $M<<N$. In particular, we can suppose that $2M<N$. Then $\sqrt{N-n}>\sqrt{N/2}, \quad\forall n\le M$.

The problem can be solved by using one more set of functions defined when $N>2M$ :

  • $H^M_M(t,N)=0, \quad\forall t,\quad 2M<N$.
  • $H^M_n(t,N)=Cn\int_0^t H^M_n(u,N)+H^M_{n+1}(u,N)du +\frac{Cn^2}{\sqrt{N/2}}t, \quad\forall n\le M-1$.

Then, when $n, M, t$ are fixed, $H^M_n(t,N)=O(\frac{1}{\sqrt{N}})$, because only linear operations (sum, integral, multiplication by something independent on $N$) are applied on $\frac{1}{\sqrt{N/2}}$.

In other words, $H^M_n(t,N)\to 0$ when $N\to +\infty$.

Lemma 4 :
Let $a, b, c\in\mathbb{R}^+$, then $\min(a,b)+c\ge\min(a,b+c)$.

Proof :

  • If $a\le b$, then $\min(a,b)+c=a+c\ge a\ge\min(a,b+c)$.
  • If $b\le a$, then $\min(a,b)+c=b+c\ge\min(a,b+c)$. $\quad\square$

Proposition 5 :
Let $n,M,N\in\mathbb{N}$ with $n<M<N/2$, and $t\in\mathbb{R}^+$.
$G^M_n(t)+H^M_n(t,N)$
$\ge \min(2, Cn\int_0^t G^M_n(u)+H^M_n(u,N)+G^M_{n+1}(u)+H^M_{n+1}(u,N)du+\frac{Cn^2}{\sqrt{N/2}}t)$.

Proof :
$G^M_n(t)+H^M_n(t,N)$
$\ge \min\big(2, Cn\int_0^t G^M_n(u)+G^M_{n+1}(u)du\big)+Cn\int_0^t H^M_n(u, N)+H^M_{n+1}(u,N)du+\frac{Cn^2}{\sqrt{N/2}}t$
$\ge \min(2, Cn\int_0^t G^M_n(u)+H^M_n(u,N)+G^M_{n+1}(u)+H^M_{n+1}(u,N)du+\frac{Cn^2}{\sqrt{N/2}}t)$
The first inequality comes from proposition 1 and the definition of H. The second inequality comes from lemma 4. $\quad\square$

By a reasoning similar to proof of proposition 2, we obtain that :

Proposition 6 :
$0\le f^N_n(t)\le G^M_n(t)+H^M_n(t,N)$ when $n<M<N/2$, and the equality holds only if $t=0$ or $f^N_n(t)=2$.

Proof
Let $M, N\in\mathbb{N}$ with $M<N/2$.
By contradiction, let's assume that proposition 6 is false for some $t>0$ and $n<M$. Since n is a natural number with an upper bound, we can take the greatest $n$ (with $M, N$ fixed) where proposition 6 is false.

To simplify the notations, I will write $GH^M_n(t,N)$ instead of $G^M_n(t)+H^M_n(t,N)$.
Let's also take $t>0$ be the first time $f^N_n(t)=GH^M_n(t,N)$. As long as it's a proof by contradiction, we also need to assume that $f^N_n(t)<2$.

We need to make sure $t$ is defined. Since $\frac{Cn^2}{\sqrt{N/2}}>\frac{Cn^2}{\sqrt{N-n}}$ and $f^N_{n+1}(0)=f^N_{n+1}(0)=0$ and the $f$ are continuous, there exists $\epsilon>0$ such that
$\forall u\in ]0,\epsilon],\quad Cn(f^N_n(u)+f^N_{n+1}(u))<\frac{Cn^2}{\sqrt{N/2}}-\frac{Cn^2}{\sqrt{N-n}}$.
It means $\forall u\in]0,\epsilon],\quad f^N_n(u)<H^M_n(u,N)\le GH^M_n(u,N)$.
So, $E:=\{u>0|f^N_n(u)=GH^M_n(u,N)\}\subset ]\epsilon,+\infty[$. Now we can take $t=\inf(E)=\min(E)$ because $f^N_n$ and $GH^M_n$ are continuous.
(In other words, we supposed by contradiction that $E\neq\emptyset$, and $f^N_n(t)<2$)

By the choice of $n$ and $t$ we have :
$f^N_n(t)\le Cn\int_0^t f^N_n(u)+f^N_{n+1}(u)du+\frac{Cn^2}{\sqrt{N-n}}t$, by the definition of $f$
$\le Cn\int_0^t f^N_n(u)+f^N_{n+1}(u)du+\frac{Cn^2}{\sqrt{N/2}}t$, because $n<M<N/2$
$< Cn\int_0^t GH^M_n(u,N)+GH^M_{n+1}(u,N)du+\frac{Cn^2}{\sqrt{N/2}}t$, because $t$ is minimal
$\le GH^M_n(t,N)$, this comes from proposition 5 (here I didn't wrote the $\min(2,\cdots)$ part because we assumed $GH^M_n(t,N)=f^N_n(t)<2$).

So, we have both $f^N_n(t)=GH^M_n(t,N)$ (definition of $t$) and $f^N_n(t)<GH^M_n(t,N)$ : contradiction. $\quad\square$

Knowing that both $G^M_n(t)\to 0$ when $M\to +\infty$ and $H^M_n(t,N)\to 0$ when $M$ is fixed and $N\to +\infty$, we conclude that $f^N_n(t)\le G^M_n(t)+H^M_n(t,N)\to 0$ when $N\to +\infty$. $\quad\square$

$\endgroup$
5
  • $\begingroup$ As it’s currently written, your answer is unclear. Please edit to add additional details that will help others understand how this addresses the question asked. You can find more information on how to write good answers in the help center. $\endgroup$ Commented Nov 17 at 15:56
  • $\begingroup$ of course, it's unclear. I needed preview. I am writing the answer, it's not finished. I've found no way to have a preview without posting. $\endgroup$ Commented Nov 17 at 15:58
  • $\begingroup$ I've temporary deleted my post and edited it while deleted (it's the only solution I currently know to see previews) and undeleted it after that. My proof is done, now. $\endgroup$ Commented Nov 19 at 2:39
  • $\begingroup$ Many thanks. Could you please specify your reasoning for the inequality $f^N_n(t)\le G^M_n(t)+H^M_n(t,N)$ when $N>2M$? $\endgroup$ Commented Nov 20 at 8:36
  • $\begingroup$ It's because $G^M_n(t)+H^M_n(t,N)\ge \min(2, Cn\int_0^t G^M_n(u)+H^M_n(u,N)+G^M_{n+1}(u)+H^M_{n+1}(u,N)du+\frac{Cn^2}{\sqrt{N/2}}t)$. So it follows almost the same inequality $f$ does. Except that here we have a $\ge$ sign, and the $\frac{Cn^2}{\sqrt{N/2}}t$ coefficient is bigger than $\frac{Cn^2}{\sqrt{N-n}}t$. I edited my proof a few minutes ago to explain it with more details. $\endgroup$ Commented Nov 20 at 16:35

You must log in to answer this question.

Start asking to get answers

Find the answer to your question by asking.

Ask question

Explore related questions

See similar questions with these tags.